PT65.S1.Q19 - last winter was mild enough to allow birds

LSATcantwinLSATcantwin Alum Member Sage
edited August 2017 in Logical Reasoning 13286 karma

https://7sage.com/lsat_explanations/lsat-65-section-1-question-19/
Help. That's all I can say. I do not see how answer choice C does anything to the argument even slightly.

Birds didn't eat at the feeders, so they got ate less and now their population is larger because of the mild winter?

Best I can do. I hate this question....

How does that do better than answer A.

Increases in bird populations sometimes occur following unusual weather patterns.

Mild winter means the winter is not as harsh as it usually is, this means that weather is unusual, and the bird population increased.

^ seems way better to me.

Comments

  • 7SageThanks7SageThanks Free Trial Member
    166 karma

    Answer choice A is just stating a correlation. Correlation alone does not necessarily support a causal conclusion.

  • 7SageThanks7SageThanks Free Trial Member
    166 karma

    I think PT 71, Section 1, #12 is a pretty similar question

  • LSATcantwinLSATcantwin Alum Member Sage
    13286 karma

    @7SageThanks said:
    Answer choice A is just stating a correlation. Correlation alone does not necessarily support a causal conclusion.

    It doesn't need to though. It just needs to strengthen the argument a bit. A correlation can strengthen an argument. It doesn't need to prove anything, just lend support. A correlation by itself isn't enough. With the facts provided though it seems to give it some credence...

  • BinghamtonDaveBinghamtonDave Alum Member 🍌🍌
    8694 karma

    I have yet to take this PT, but I took a look at this question briefly in order to assist you.
    We are trying to strengthen the reasoning in the argument. One of the ways that we can do that is by making the existing premises more relevant to the conclusion. I like to think of this particular path of strengthening the argument as essentially a method of emboldening a premise in such a way that it lends more support to the conclusion than it initially did. As an aside, I think it is possible for an experienced test taker to actually see a loose-end in a premise for a strengthening question and keep it in mind when we head to the answer choices. I hope that makes sense by the end of this explanation.

    What we are trying to strengthen here is the idea that last years mild winter "is responsible" (meaning it caused) the larger bird population. How did we get to that conclusion?
    Well, we are told that the mild winter allowed the birds to:
    -forage naturally
    and
    lessen the attrition of flying south
    Attrition taking on the definition here of wearing down or exhausting

    So, there is our core argument:
    The weather was nice, birds could forage in the fields and meadows etc
    and they didn't have to fly all the way to Florida: which is an exhausting journey:
    therefore, the weather was responsible for the increase in their population this year.

    So there are several "traditional" ways of strengthening this argument: we could say for instance that the migration to Florida accounts for the vast majority of the usual population drop and that absent that cause, we can support our conclusion. We could say that foraging naturally provides the birds with a level of vitamins and minerals that is unparalleled and contributes heavily to the maintenance of their general health: possibly building up their immune system.

    Instead, this argument took a small (seemingly irrelevant) throwaway line from the stimulus and made it relevant to the conclusion! This question says: hey, test taker: do you remember when we told you that the birds actually use the bird feeders less? Well actually: using bird feeders less, actually lesses the time that the birds are in a spot where predators can often eat them! In short, less birds are being eaten at bird feeders, because less are feeding at bird feeders because they are foraging naturally because the weather is nice.

    The inclusion of answer choice (C) actually takes (what I see anyway) as a throwaway line from the stimulus and fashions it into a fact that helps support the conclusion.

    I hope this helps: my tip moving forward is to start compiling instances where a seemingly irrelevant/throwaway line in a stimulus can be fashioned into something that can strengthen the argument.

    David

  • LSATcantwinLSATcantwin Alum Member Sage
    edited August 2017 13286 karma

    @BinghamtonDave

    Okay that all made 100% sense.

    I still don't see why A is wrong though. They both seem to make small leaps to strengthen.

    A; Sometimes a bird populations increase with unusual weather.

    We had a mild winter, therefore the weather was different, therefore winter = increase in pop

    C; Birds don't feed at feeders; therefore Less birds get ate by preds, therefore the bird population is bigger than normal and that's due to the winter.

    Like what if say, 300 birds of 50,000, feed at the feeders usually. Now 200 do. Can we really say that its a "larger-than-usual population" now?

    Edit:
    That's even assuming all 300 get ate, maybe only 10 of the 300 do normally!

  • BinghamtonDaveBinghamtonDave Alum Member 🍌🍌
    8694 karma

    What we are asked to do is most strengthen the argument here. What (A) tells us is that “sometimes” when have the effect we see above, we have the cause we see above. But it doesn’t actually say that: (A) says sometimes we have the effect we see above with the cause of what we have to assume is a super-set containing the cause we have above: “unusual weather” vs. “mild winter.”
    Let’s table the idea that these might be different concepts: which I believe is a basis some would disqualify this answer choice on. The vagueness of (A) with “unusual weather pattern” could mean frost in May. Granting the idea that the mild winter we have above in the stimulus is in fact “unusual,” what does (A) actually tell us?

    -Are there any other factors? Meaning, was there anything else that could have caused the increased population? (A) is just a weak correlation: read weakly we have “one time there was a increased bird population after an unusual weather pattern.”
    -In the stimulus above, are we operating in the “sometimes” area of this answer choice? Or are we in the coincidence area outside of the sometimes area of the answer choice?
    -How much does it really strengthen the conclusion of: “mild winter caused the increase in population in birds” to say “sometimes mild winters cause increase populations in birds.”

    I will concede that if read generously, and with great care (A) does strengthen the argument, but not anywhere close to (C). (C) tells us that the shift in behavior due to weather has placed the birds in a collectively less dangerous spot to their lives. Less dangerous means more survive, which is only a small leap away from saying that the numbers grow because less are dead.

    (A) is not irrelevant, but it just doesn’t have what we should be looking for in a strengthening question like this one in my estimation. We see variations of (A) often in weakening questions contain a similar logical structure as we see above.

    I want to leave you with 2 questions that I had on my mind when I typed this up, I hope they are helpful to you and anyone reading this thread:
    The first is a strengthening question that takes an unused premise and makes it relevant to the conclusion, much like we see above: 44-4-14, always be on the look out for a premise left sitting there in strengthening questions: this is an opening that we have to be sensitive to possibly using. Take this a step further and not only be on the look out for the use of a premise left sitting there, but be on the lookout for the use of the premise left sitting there in a roundabout/vague/very LSAT-ish way lol! The test writers love to do that!

    The next question is one of the rarest questions I have come across in my studies and a question that I would probably get wrong 7-8 times out of 10. It is an example of how the LSAT plays on the word "most" in a question stem, sometimes the "most" of a group is something I'm not entirely comfortable with: PT C2-3-15

    This exchange on 65-1-19 has been fruitful for me.

    David

  • LSATcantwinLSATcantwin Alum Member Sage
    13286 karma

    @BinghamtonDave it's helped me too. I like the idea of targeting lose ends on questions such as this one. I'll use that strategy next time. I can say I wasn't 100% happy with A when I picked it. I was able to justify it though, and that is dangerous. A correct answer might be there and I may not be able to justify it. At least seeing the gap like this will help me out. Thank you

Sign In or Register to comment.